Solving Multivariable Limits: Tips & Tricks

In summary, the conversation discusses two limit problems, one involving x and y approaching 0 and the other involving x and y approaching (1,1). They also mention using the paths y=x^3 and y=x to approach the limits, but it is determined that the limit for the bottom problem does not exist because the function becomes large without bound. The limit for the top problem can be solved using polar coordinates and the squeeze theorem.
  • #1
Wesleytf
32
0
I'm taking multi-variable after having a while off from school, so forgive me if these are simple ones that I just don't "see"

Homework Statement


lim (x, y) --> 0, 0 [tex]\frac{x^2 y^2 e^y}{x^4+4y^2}[/tex]

and

lim (x, y) --> (1, 1) [tex]\frac{x-y}{x^3-y}[/tex]

Homework Equations


The Attempt at a Solution



The bottom one I feel doesn't exist because as x->0+, the ^3 is making it larger, where as when x->0-, the ^3 is making it smaller. I know this is poor logic; it's just a gut feeling about it. substituting y=mx or similar didn't get me anywhere. Plugging obviously doesn't work. I don't see anyway to simplify, but maybe there is a way. I also don't think polar coordinates will work for either.

I really think I only need a hint to the method of solution, so don't go solving the whole thing for me.
 
Last edited:
Physics news on Phys.org
  • #2
For the bottom one, suppose you approach along the path y=x^3? For the top one you don't have to worry about the factor e^y, why not? Can you handle the rest?
 
  • #3
Dick said:
For the bottom one, suppose you approach along the path y=x^3? For the top one you don't have to worry about the factor e^y, why not? Can you handle the rest?

For the top one, e^y -->1 as y->0. I wasn't sure if saying that and then doing the rest of the limit was a legal move. Should be easy now (I see one way by polar coordinates and then squeeze theorem, I think)

For the bottom one, I tried that, but doesn't that just 'break' the function--as in, we can't tell what it will be by that method? The way I thought the "two different limits for two different paths" property worked was that it would only work if you found two actual different paths. Taking y=x^3 makes one path not exist; is that enough to make the limit of the function not exist?
 
  • #4
Wesleytf said:
For the top one, e^y -->1 as y->0. I wasn't sure if saying that and then doing the rest of the limit was a legal move. Should be easy now (I see one way by polar coordinates and then squeeze theorem, I think)

For the bottom one, I tried that, but doesn't that just 'break' the function--as in, we can't tell what it will be by that method? The way I thought the "two different limits for two different paths" property worked was that it would only work if you found two actual different paths. Taking y=x^3 makes one path not exist; is that enough to make the limit of the function not exist?

If the function has a limit, it has to approach that limit along all paths. If the function doesn't even exist along a path, then the limit doesn't exist. If you think about what's happening 'close' to the path y=x^3 then you'll see the function becomes large without bound.
 
  • #5
Dick said:
If the function has a limit, it has to approach that limit along all paths. If the function doesn't even exist along a path, then the limit doesn't exist. If you think about what's happening 'close' to the path y=x^3 then you'll see the function becomes large without bound.
ha, I had already went back and put it on paper, and as soon as I did the behavior 'close' to y=x^3 became clear. thanks! hopefully my brain will uncrustify itself soon...
 
  • #6
follow up on the bottom one: Subbing y=x^3 will not work because it is not within the definition of a limit. However, comparing the substitutions y=x and y=1 does work to prove that it indeed DNE.
 

Related to Solving Multivariable Limits: Tips & Tricks

1. What is a multivariable limit?

A multivariable limit is a mathematical concept that represents the behavior of a function as it approaches a point in a multi-dimensional space. It is used to determine the value of a function at a specific point, by examining the behavior of the function as it approaches that point from different directions.

2. How do I solve a multivariable limit?

To solve a multivariable limit, you can use various methods such as direct substitution, factoring, or using trigonometric identities. It is important to also consider the different paths of approach to the point and evaluate the limit from each path to ensure that the limit exists.

3. What are some common techniques for solving multivariable limits?

Some common techniques for solving multivariable limits include using L'Hopital's rule, using algebraic manipulation, and using trigonometric identities. It is also helpful to visualize the function and the point of interest in order to determine the appropriate method to use.

4. How do I know if a multivariable limit exists?

A multivariable limit exists if the limit evaluated from all paths of approach to the point is equal. This means that the behavior of the function is consistent from all directions and there is a well-defined value at the point of interest. If the limit is different for different paths of approach, then the limit does not exist.

5. Can I use a calculator to solve multivariable limits?

While calculators can be helpful in evaluating multivariable limits, they are not always accurate and should not be solely relied on. It is important to understand the concept and use appropriate techniques to solve multivariable limits, rather than relying solely on a calculator.

Similar threads

  • Calculus and Beyond Homework Help
Replies
7
Views
740
  • Calculus and Beyond Homework Help
Replies
10
Views
867
  • Calculus and Beyond Homework Help
Replies
5
Views
945
Replies
9
Views
796
  • Calculus and Beyond Homework Help
Replies
8
Views
859
  • Calculus and Beyond Homework Help
Replies
7
Views
964
  • Calculus and Beyond Homework Help
Replies
24
Views
2K
  • Calculus and Beyond Homework Help
Replies
4
Views
762
  • Calculus and Beyond Homework Help
Replies
8
Views
1K
  • Calculus and Beyond Homework Help
Replies
13
Views
2K
Back
Top